Supremums 'alpha' and 'beta' problem

  • Thread starter Thread starter kmeado07
  • Start date Start date
  • Tags Tags
    Alpha Beta
Click For Summary
SUMMARY

The discussion centers on proving that the supremum of the set A+B, formed by the addition of two non-empty sets of real numbers A and B with supremums α and β, is equal to α + β. The proof utilizes the completeness axiom of real numbers, establishing that α + β serves as an upper bound for A+B. By employing proof by contradiction, it is shown that if α + β were not the least upper bound, it would lead to a contradiction, confirming that sup(A+B) = α + β.

PREREQUISITES
  • Understanding of supremum and infimum in real analysis
  • Familiarity with the completeness axiom of real numbers
  • Knowledge of proof techniques, particularly proof by contradiction
  • Basic operations with sets, specifically addition of sets
NEXT STEPS
  • Study the completeness axiom in more detail and its implications in real analysis
  • Learn about the properties of supremums and infimums in various mathematical contexts
  • Explore advanced proof techniques, including indirect proofs and their applications
  • Investigate the concept of set operations and their effects on supremums
USEFUL FOR

Mathematics students, particularly those studying real analysis, educators teaching set theory, and anyone interested in the properties of supremums and their proofs.

kmeado07
Messages
40
Reaction score
0

Homework Statement



Let A, B,be two non empty sets of real numbers with supremums 'alpha' and 'beta' respectively, and let the sets A+B and AB be defined by: A+B={a+b / a belongs to A, b belongs to B}, AB= {ab / a belongs to A, b belongs to B}.
Show that alpha+beta is a supremum of A+B.

Homework Equations





The Attempt at a Solution



Im not really sure how to go about this question. Maybe use the completness axion, all real numbers satisfy it, which means that A+B must have a supremem...
 
Physics news on Phys.org


To show that alpha+beta is the supremum of A+B, use the definition of supremum.
 


a = supA -> a>=x1 when x1 in A
b = supB -> b>=x2 when x2 in B

a+b>=x1+x2 -> x1+x2 in A+B -> a+b is an upper bound.
thus sup(A+B) <= a+b.
lets say a+b-sup(A+B)=epsilon
which is , sup(A+B)=a+b-epsilon

there is x in A, which is smaller than a, and x=a-epsilon/2
there is also y in B, which is smaller than a, and y=b-epsilon/2
there is x1 in A which is bigger than x (cuz x is smaller than a [which is supA]) and there is y1 that is bigger than y too. if that so, we can say :
x1 > x -> x1>a-epsilon/2
y1>y -> y1>b-epsilon/2
x1+y1 is in A+B and is also
x1+y1 > a+b-epsilon = sup(A+B)
contrary to the definition of sup(A+B). in conclusion,
sup(A+B)=a+b=supA+supB
 


Well the definition of supremum is that it's the least upper bound, it is greater than or equal to each element in the set.

I still don't know how so start off showing that alpha+beta is the sup of A+B...
 


i probably haven't made myself clear. first of all, i have shown that supA+supB is an upper bound of the set A+B :
supA+supB >= sup(A+B)
later, I've proofed by contradiction that supA+supB>sup(A+B) so I've assumed that there is a number, epsilon, which is bigger than zero, that :
sup(A+B)=supA+supB-epsilon. and that has brought me to the conclusion that sup(A+B)=supA+supB.
 
Last edited:


If "x" is any member of A+ B, then a= a+ b for some a in A, b in B. Since \alpha is an upper bound on A, x\le \alpha. Since \beta is an upper bound on B, b\le\beta. Therefore, a+ b\le ? That shows that \alpha+ \beta is an upperbound on A+ B.

Now you need to show it is the smallest upper bound and I recommend you use an "indirect proof" or "proof by contradiction" to do that: suppose \alpha+ \beta is NOT the least upper bound of "A+ B". Suppose there exist some lower bound, \gamma smaller than \alpha+ \beta. Let \epsilon= (\alpha+ \beta)- \gamma. Can you find some "a" in A so that x&gt; \alpha+ \epsilon/2? Can you find some "b" in B so that y&gt; \beta+ \epsilon/2? If so, what can you say about a+ b?
 
Question: A clock's minute hand has length 4 and its hour hand has length 3. What is the distance between the tips at the moment when it is increasing most rapidly?(Putnam Exam Question) Answer: Making assumption that both the hands moves at constant angular velocities, the answer is ## \sqrt{7} .## But don't you think this assumption is somewhat doubtful and wrong?

Similar threads

  • · Replies 5 ·
Replies
5
Views
2K
Replies
2
Views
2K
  • · Replies 3 ·
Replies
3
Views
2K
  • · Replies 1 ·
Replies
1
Views
2K
  • · Replies 4 ·
Replies
4
Views
2K
  • · Replies 8 ·
Replies
8
Views
8K
  • · Replies 4 ·
Replies
4
Views
1K
  • · Replies 10 ·
Replies
10
Views
8K
  • · Replies 3 ·
Replies
3
Views
1K
  • · Replies 2 ·
Replies
2
Views
2K